Magnitudes, Resultant Force (missing angle)

In summary, the person is trying to solve an equation for an angle involving a weight, F3, and two other weights, F1 and F2. They are able to solve the equation if they can find the phi value for F3. They are unable to find the phi value for F3, so they are stuck. However, they are able to find the angle beta using the sine and cosine values for F3 and the information in the picture.
  • #1
jegues
1,097
3

Homework Statement


See attachement.


Homework Equations





The Attempt at a Solution



I would be able to solve it if I could somehow find [tex] \phi [/tex] to describe the angle of F3 relative to the positive x axis. Can anyone see how to solve that specific angle?

Then and can simply sum as follows

[tex]
F_{x} = F1cos(\theta) + F2cos(\alpha) + F3cos(\phi)[/tex]

and

[tex]
F_{y} = F1sin(\theta) + F2sin(\alpha) + F3sin(\phi)[/tex]

and
[tex]
F = \sqrt{F_{x}^{2} + F_{y}^{2}}[/tex]

Then let [tex]\beta[/tex] be the resultant angle,

[tex] \beta = tan^{-1}(\frac{F_{y}}{F_{x}} ) [/tex]
 

Attachments

  • Q2.9.JPG
    Q2.9.JPG
    10.4 KB · Views: 605
Physics news on Phys.org
  • #2
You don't need to know the angle [itex]\phi[/itex] itself, just its sine and cosine. And the picture gives x and y sides of a similar triangle to the one with F3 as the hypotenuse.
 
  • #3
the picture gives x and y sides of a similar triangle

I think you are thinking of the similar triangle for F2 not F3. Am I correct in my assumption?
 
  • #4
No, the diagram gives you similar information for F3. It's just not explicitly drawn in like it is for F2. F3 lies on the hypotenuse of a triangle in the picture. You should be able to identify the lengths of the legs of that triangle.
 
  • #5
Well it can't be the triangle with the sides 0.2 and 0.3 because there is a corner that intersects it at the bottom right. I don't see how the 0.1 near the joint helps us either.

Is it safe to assume that the corner in the bottom right doesn't effect our triangle so it will have sides of 0.2 and 0.3?

Otherwise, I'm just not seeing it! Any more help?
 
  • #6
jegues said:
Well it can't be the triangle with the sides 0.2 and 0.3 because there is a corner that intersects it at the bottom right. I don't see how the 0.1 near the joint helps us either.

Is it safe to assume that the corner in the bottom right doesn't effect our triangle so it will have sides of 0.2 and 0.3?

Otherwise, I'm just not seeing it! Any more help?

Truth is, I didn't see that little corner so you are correct. But I would bet money that it is safe, and you are expected, to ignore it, and use the .2 and .3.
 
  • #7
Is there any other way I could solve that angle without using similar triangles (using sides 0.2 and 0.3)? If so how?

If there's no other way, then I guess I'm stuck assuming it's the safe(and correct) way to do it.
 
  • #8
I don't see why the corner is a problem.
 
  • #9
Because if that corner is there then the sides aren't going to be 0.2 and 0.3 respectively.
 
  • #10
I just looked at the picture again. The corner is not a problem at all. It is only the dimension arrows that give it that appearance. The dimensions are OK as given.
 
  • #11
jegues said:
Well it can't be the triangle with the sides 0.2 and 0.3 because there is a corner that intersects it at the bottom right.
The thin lines are all dimension lines, so that is not a "corner." Look at the 0.3m measurement: there are arrows pointing up and down. The arrow pointing up points to a dimension line, as does the arrow pointing down. What you're seeing as a "corner" is this lower dimension line intersecting with the right-hand dimension line for the 0.2m measurement. The thin line extending out of the bold 1200N force line is just an extension of the force line which is being used to give us the slope of the force.

jegues said:
WI don't see how the 0.1 near the joint helps us either.
There are a couple of measurements that don't relate to this problem. My guess is that this diagram comes from a book with several questions relating to it.
 
  • #12
jegues said:
Because if that corner is there then the sides aren't going to be 0.2 and 0.3 respectively.
Draw a vertical line from the corner to the upper horizontal line. The vertical line, the upper horizontal line, the horizontal edge of the corner, and the measurement arrows form a rectangle. The opposite sides of the rectangle are the same length, so the vertical line is 0.3 m. Similarly, the horizontal distance from where F3 acts to the corner is 0.2 m.
 
  • #13
Thank you for the clarification, I seem to have forgot that we were only using the similar triangle to solve the angle for F3 in relation to the positive x-axis.

I've solved the problem now!
 

FAQ: Magnitudes, Resultant Force (missing angle)

1. What is the definition of magnitude?

Magnitude refers to the size or amount of something, such as the strength or intensity of a force.

2. How is magnitude measured?

Magnitude can be measured using various units, depending on the quantity being measured. For example, force is typically measured in Newtons (N), while energy is measured in joules (J).

3. What is a resultant force?

A resultant force is the overall force that results from the combination of two or more individual forces acting on an object. It is the vector sum of these forces, taking into account their direction and magnitude.

4. How do you calculate the resultant force?

The resultant force can be calculated using the Pythagorean theorem or trigonometric functions. In the case of missing angle, you can use the law of cosines to find the magnitude of the resultant force and the law of sines to find the missing angle.

5. Why is understanding magnitudes and resultant forces important in science?

Magnitudes and resultant forces are essential in understanding and predicting the behavior of objects and systems in the physical world. They are crucial in fields such as mechanics, engineering, and physics, and are used to analyze and design structures, machines, and other systems.

Back
Top